Difference between revisions of "2002 AMC 12A Problems/Problem 2"

Line 1: Line 1:
 +
{{duplicate|[[2002 AMC 12A Problems|2009 AMC 12A #2]] and [[2002 AMC 10A Problems|2009 AMC 10A #6]]}}
 +
 
==Problem==
 
==Problem==
  
Line 8: Line 10:
 
==Solution==
 
==Solution==
  
<math>\dfrac{x-9}{3}=43</math>
+
We work backwards; the number that Cindy started with is <math>3(43)+9=138</math>. Now, the correct result is <math>\frac{138-3}{9}=\frac{135}{9}=15</math>. Our answer is <math>\boxed{\text{(A)}\ 15}</math>.
 
 
<math>x=43*3+9=138</math>
 
 
 
<math>\dfrac{x-3}{9}=15 \Rightarrow \mathrm {(A)}</math>
 
  
 
==See Also==
 
==See Also==
  
 
{{AMC12 box|year=2002|ab=A|num-b=1|num-a=3}}
 
{{AMC12 box|year=2002|ab=A|num-b=1|num-a=3}}
 +
{{AMC10 box|year=2002|ab=A|num-b=5|num-a=7}}

Revision as of 07:45, 18 February 2009

The following problem is from both the 2009 AMC 12A #2 and 2009 AMC 10A #6, so both problems redirect to this page.

Problem

Cindy was asked by her teacher to subtract 3 from a certain number and then divide the result by 9. Instead, she subtracted 9 and then divided the result by 3, giving an answer of 43. What would her answer have been had she worked the problem correctly?

$\mathrm{(A) \ } 15\qquad \mathrm{(B) \ } 34\qquad \mathrm{(C) \ } 43\qquad \mathrm{(D) \ } 51\qquad \mathrm{(E) \ } 138$


Solution

We work backwards; the number that Cindy started with is $3(43)+9=138$. Now, the correct result is $\frac{138-3}{9}=\frac{135}{9}=15$. Our answer is $\boxed{\text{(A)}\ 15}$.

See Also

2002 AMC 12A (ProblemsAnswer KeyResources)
Preceded by
Problem 1
Followed by
Problem 3
1 2 3 4 5 6 7 8 9 10 11 12 13 14 15 16 17 18 19 20 21 22 23 24 25
All AMC 12 Problems and Solutions
2002 AMC 10A (ProblemsAnswer KeyResources)
Preceded by
Problem 5
Followed by
Problem 7
1 2 3 4 5 6 7 8 9 10 11 12 13 14 15 16 17 18 19 20 21 22 23 24 25
All AMC 10 Problems and Solutions